In which number does the 7 represent 10 times the value it represents in 167,300

Answers

Answer 1

Answer:

I'm not totally sure, but I think the answer is thousands.

Step-by-step explanation:


Related Questions

The table shows the number of tickets sold and the profit made at the fundraiser

2 $18
4 $36
6 $54
8 $72

Which describes the relationship between the number of tickets sold and the profit

Each ticket sold yields $8 of profit
Each ticket sold yields $9 of profit
Each ticket sold yields $16 of profit
Each ticket sold yields $18 of profit

Answers

Answer:

first one is The table shows the number of tickets sold and the profit made at a fundraiser. Tickets Sold Profit

2 $18

4 $36

6 $54

8 $72

Which describes the relationship between the number of tickets sold and the profit?

Each ticket sold yields $8 of profit.

Each ticket sold yields $9 of profit

. Each ticket sold yields $16 of profit.

Each ticket sold yields $18 of profit.

Step-by-step explanation:

done

Answer:

b

Step-by-step explanation:

mops and buckets are sold in a shop. 9 mops cost £72. the ratio of the cost of a mop to the cost of a bucket is 4:7. work out the cost of 6 buckets​

Answers

Answer: 52 pound because if it’s 4:7 7 add 4 = 11 divide that by 72 gets you 52

A coin is flipped 160 times. It landed on heads 82 times and tails 78 times. A different coin is flipped 250 times. It landed on heads 115 times and tails 135 times. What is the empirical probability of flipping a heads on the first coin AND flipping a heads on the second coin

Answers

Answer:

a. first coin heads: 82/160

b. second coin heads: 115/250

a AND b: a x b = 82/160 x 115/250

Step-by-step explanation:

if not then im confuzzled

PLZ HELPPPPPPPPPPPPP​

Answers

Answer:

a, d

Step-by-step explanation:

? Help me plzzz correct answer gets brainliest

Answers

Answer:

B?

Step-by-step explanation:

im s0rry im n0t sure

PLease tell me how to do it that will give you brainliest

Answers

Answer: Table H would be the correct answer;

The rule of a function is that for each x-value given there can't be more than 1 y-value

In Table F:

x = -13, then y = -2

x = -13, then y = 0

x = -13, then y = 5

x = -13, then y = 7

For the x-value -13, there are 4 different y-values, so it's not a function.

In Table G:

x = -6, then y = 3

x = -1, then y = -1

x = -1, then y = 5

x = 10, then y = -9

For the x-value -1, there are 2 different y-values, hence this isn't a function.

In Table H:

x = 1, then y = 4

x = 3, then y = 4

x = 7, then y = 4

x = 12, then y = 4

For each x-value, there is only 1 y-value, so this is a function.

In table J:

x = -9, then y = -7

x = -2, then y = -5

x = 0, then y = 0

x = 0, then y = 6

For the x-value 0, there are 2 different y-value therefore this isn't a function

Hope this helps!

The radius of a circle is 4 feet. What is the circle's circumference?Use 3.14 for
r=4 ft

Answers

Answer:

50.24 is the circumfrence

If we were to use actual pi the answer would be -

50.26548246

Answer:

25.12

Step-by-step explanation:

2pi4=2*3.14*4=25.12

conviince me for my question (brainliest plzz)

What is this shape? (sorry I'm d.umb)

Answers

Answer:

triangular prism

Step-by-step explanation:

repost on my warm up plz help

Answers

Answer:

oh srry ur not a scam

Step-by-step explanation:

Answer:

#1: (Problem 3² × 3³, Answer: 243) (Problem 3⁵, Answer: 243), yes the answer matches

#2: (Problem 2³ × 2⁴, Answer: 128) (Problem 2¹², Answer: 4096), No, the answer doesn't match.

#3: (Problem (-2)⁴ × (-2)², Answer: 64) (Problem (-2)⁸, Answer: 256), No the answer doesn't match.

#4: (Problem (-3)² × (-3)⁵, Answer: -2187) (Problem (-3)⁷, Answer: -2187), yes the answer matches

The pattern is: When 2 whole numbers raised to a power are multiplied together, the base stays the same and the exponent add up

Hope this helps!

Water is coming out of a fountain is modeled by the function f(x)=-x^2+8x+2 where f(x) represents the height, in feet, if the water from the fountain at different times x, in seconds

Answers

The answer would be 1

HELP 30 POINTS please in a hurry
Is the relation shown below a function? Use the graph below to justify your response.
(0, 0), (2, 0), (2, 2), and (3, 4)

Answers

To be a function every input value (x) can only have one output value (y)

On the graph there are 2 out put values for x = 2 so it is not a function.

The answer would be C.

please simple answers i had state testing today

Answers

Answer:

The mode is 10. The range is 6. The IQR is 5.

Step-by-step explanation:

A volume of the cone is (1/3) the volume of a cylinder. The volume of the sphere is what fraction of the volume of the cylinder?

Answers

Answer:

4/3

Step-by-step explanation:

To know this, let's write down the formulas for the volume of cylinder and sphere.

Vs = 4/3πr³   (1)

Vc = π r² h    (2)

Now, we do have a little problem here and its the fact that the sphere do not have a height like the cylinder do. But in this case so if you want to have an idea of the fraction of the volume, we will assume that the cylinder has the same height as its radius. Assuming this we have the following:

Vs / Vc = 4πr³ / 3πr²h

Vs/Vc = 4πr³ / 3πr³

From here, we can cancel out the values of π and r³:

Vs/Vc = 4/3

Vs = 4/3 Vc

Thus we can conclude that the volume of the sphere is 4/3 the volume of a cylinder.

Hope this helps

The amount that a craftsman will charge for an item depends on the
amount that he spends on materials, m, and is determined by the
equation a = 2.5 + 2.5m, where a is the amount charged for each
item. What is the value of the dependent variable, if the value of the
independent variable is 3?

Answers

Answer:

$10

Step-by-step explanation:

m is the independent variable, and a the dependent one.

If m = 3, a = 2.5 + 2.5(3) = 10

The craftsman charges $10 when m = 3.

Identify the net that corresponds to the figure below. A. B. C. D.

Answers

Answer:

The net that corresponds to it is ,B

Unit l2 trigonometry homework 6 law of cosines
Does anyone know the answer please help!​

Answers

Step-by-step explanation:

The law of cosine states that:

[tex]C^2=B^2+A^2-2AB\cos(\theta)[/tex], where C is the side opposite of angle theta, and A and B are the adjacent sides of the angle.

In this case, A is equal to 21, and B is equal to 30 (or it could also be the other way around). That means that:

[tex]C^2=21^2+30^2-2\cdot21\cdot30\cos(73)\\C=\sqrt{21^2+30^2-2\cdot21\cdot30\cos(73)}=\boxed{31.186722}[/tex] (to nearest 6 digits)

Answer:

102.61

Step-by-step explanation:

assuming that QS is the hypothesis we us the fromula like this:

[tex]30cos(73)[/tex]

which gives us 8.7711511417 or we can round to 8.77

assuming that QS is the adjacent  we use:

[tex]\frac{30}{cos(73)}[/tex]

which gives us 102.609108595 or rounded to 102.61

looking at both answers we can see that 102.61 would be the correct answer.

Question 2
0.5 pts
Write 0.1133 as a fraction.
(Does not need to be simplified)
[a]
Question 3
0.5 pts
what

Answers

Answer:

1133/10000

Step-by-step explanation:

Find the volume of a cylınder whose base is a circle with a diameter of 18 inches.

Answers

Answer:

257.48 inches

Step-by-step explanation:

pi * radius^2 * Hight

find radius by dividing the diameter in half

pi = 3.14

The volume of the figure, round to the nearest hundredth.

Answers

Answer: Imma need a picture to find the volume love.

Step-by-step explanation:

If you place a 32-foot ladder against the top of a building and the bottom of the
ladder is 24 feet from the bottom of the building, how tall is the building? Round to
the nearest tenth of a foot.
Please respond ASAP

Answers

Answer:21.2

Step-by-step explanation:

delta math

Which statement correctly describes the expression (-15)3?
A. One factor is positive and one factor is negative, so the product
will be negative
B. Both factors are positive, so the product will be positive.
C. One factor is positive and one factor is negative, so the product
will be positive.
D. Both factors are negative, so the product will be positive.

Answers

The answer is A.

-15 is a negative factor

3 is a positive factor

When a negative and positive is multiplied it produces a negative answer

PLEASE HELP ME I AM BEING TIMED, I WILL MARK BRAINLIEST FOR WHOEVER GETS THIS RIGHT, WITH AN EXTRA 10 POINTS.

Answers

Answer:

Yes table 1 is

not proportional for table 2

Step-by-step explanation:

6 for table 1

nothing for table 2

find the cost of 24 piece of shshi from each restaruant

Answers

Answer:

sushi* and there is no picture.

Step-by-step explanation:

Jack wants to purchase a DVD player who’s price was $162.50 he has $50 only how much more money does Jack need to purchase the DVD player

Answers

$112.50 is how much jack will need
Jack needs $112.50 to purchase the DVD player

$162.50- 50= 123.50

if 5a + 9 = 5 + 3a, then 2a=​

Answers

Step-by-step explanation:

5a+9=5+3a

2a=-4 is the required value

One evening, Mr. Smith, Sam, and Sydney went to see the Myrtle Beach Pelicans’ Game. They each bought a ticket and a hat that cost $10 each. How much was each ticket to enter the ballpark?

Answers

Answer:

You can't answer this question without knowing the total amount of money they all had in total.

What is the quotient of 5.76 x 10^7 and 9.0 x 10^4 expressed in scientific notation?

Answers

Answer:

6.4 x 10²

Step-by-step explanation:

1st step: divide 5.76 by 9 which is .64

2nd step: 10^7÷10^4 = 10^7-4 or 10³

What is a reasonable domain and range for the function in #13 If the frame must be at least 1 inch and no more than 3 inches wide

Answers

Answer:

[tex]Domain: \{1 \le x \le 3\}[/tex]

Step-by-step explanation:

Given

See attachment for options

Required

Determine the domain and the range

From the question, we understand that:

[tex]x \ge 1[/tex] --- at least 1 inch

[tex]x \le 3[/tex] --- not more than 3 inches

So, the domain is:

[tex]x \ge 1[/tex] and [tex]x \le 3[/tex]

Rewrite as:

[tex]1 \le x[/tex] and [tex]x \le 3[/tex]

Combine both:

[tex]1 \le x \le 3[/tex]

Hence, the domain is:

[tex]Domain: \{1 \le x \le 3\}[/tex]

There is no available to determine the range.

A general rule is that, substitute the limits of the domain (i.e. 1 and 3) in the function formula, the result is the range of the dataset.

For instance:

[tex]A(x) = 5x[/tex]

We have:

[tex]A(1) = 5*1=5[/tex]

[tex]A(3) = 5*3=15[/tex]

The range will be:

[tex]Range: \{5 \le A(x) \le 15\}[/tex]

Hi could some PLEASE help me !!!!

Answers

Answer:

3x=78+x-2

Step-by-step explanation:

please help whoever wins gets brainliest

Answers

Looks like 108 because if we add the missing square the big square would be 12*12 or area equal to 144. We subtract the missing square which is 6 by 6 or 36 area we get 108 ft.
Another way would be cutting it into a rectangle and square.
Other Questions
A cell phone box in the shape of a rectangular prism is shown. The height of the box is 4 cm.The height of the original box will be increased by 3.5 centimeters so a new instruction manual and an extra battery can be included. Which is closest to the total surface area of the new box?A) 707 square centimetersB) 479 square centimetersC) 959 square centimetersD) 738 square centimeters Please factor using the x method ( please do not answer if you don't know how to do it ) Question 8 of 10Fill in the blank in the following dialogue with the appropriate verb below.-Elle ____ sa voiture?- Oui, elle en achte une neuve.A. vendB. perdC. dependD. attendSUBMIT The type of stem cells that are thought to be pluripotent are found inA.) embryosB.) developed tissuesC.) cord bloodD.) the placenta. please help me with this question!!! A litter of kittens consists of two grayfemales, one gray male, two blackfemales, and two black males. Yourandomly pick one kitten. The kitten isgray or male. How does this example support the author's main purposeill the book?denO. It shows how citizens' actions affect today's issues. Find the surface area of the regular pyramid.PLS HELP ME Mi novio y yo nosel verano pasado.conocimosconocamos hey! ill give brainliest please help! QUESTION 2.Trevor bought new sneakers originally listed for $64.99. They are 15% off. Which equation can be usedto find the amount Trevor will save?A. X= 1.15(64.99)B. x = 15(64.99)C. x = .85(64.99)D. x = .15(64.99) Can anyone help me with my homework A substance has a pH of 4. Is it an acid or a base? Tim earns $52.50 for 5 hours of dog walking. He earns the same amountper hour each time he dog walks. How many hours does Tim need to dogwalk to earn $168? find the coordinates of point p along the direct line segment ST so that SP to PT is 1 to 3. S= (-6,7) and T=(9,25) [tex] \sqrt[2]{a} {}^{2} [/tex] Factor 32x15 . If the expression cannot be factored, write cannot be factored. Help please someone please what is the answer Present Subjunctive - Adjective Clauses: Practice 1 Instrucciones Cada una de las oraciones contiene una clusula adjetival pero las oraciones no estn en el orden correcto. Cambia el orden de las palabras para que la oracin tenga sentido. /133 1. gerente busca sepa El a hablar que espaol. alguien 2. sea que mis sueldo pagar para suficiente un Necesito deudas. 3. alcanzables. sean fijar que deben estudiantes Los metas 4. accin. que Quiero pelcula ver una tenga mucha 5. dedicados. compaa empleados existe no que tenga No una exitosa 6. busca se una universidad que Juan en ciencias. especialice 7. No aos. menos haga que conozco a en de 2 la maestra nadie 8. buscan que que jefes tengan mejores empleados nuevos a Los empleados pasados. habilidades 9. Marta el que ahora. aun no ms quiere cargos trabajo cumplir un difciles que tiene que tenga de 10.en que tienes No mejor hay ahora. paguen trabajo que ciudad que el esta trabajos 11 octo algunas comestre fciles on matricularnos What do you notice about the narrative voice and viewpoint in the novel To Kill A Mockingbird?